Merge branch 'master' of git+ssh://stbuehler.de:522/qm1-script

This commit is contained in:
Oliver Groß 2011-07-21 14:02:16 +02:00
commit cc42cb882c
10 changed files with 468 additions and 327 deletions

2
.gitignore vendored
View File

@ -4,4 +4,4 @@
*.toc
*.out
theo2.pdf
theo2.kilepr

View File

@ -6,6 +6,7 @@
% undendlich dim. raum
% was zum henker ist ein operator
% impulsoperator
%definition bedingte wahrscheinlichkeiten
\chapter{Notationen}
\section{Dirac-Notation}

View File

@ -298,10 +298,10 @@ $\rightarrow$ $B \ket{n,r}$ liegt im Untrraum zu $a_n$
\end{equation}
\item[Fall (2)] $a_n$ entartet
\begin{equation}
\bra{m,s} \cdot B \cdot \ket{n,r} = B_{s,r}^{(n)}
\bra{n,s} \cdot B \cdot \ket{n,r} = B_{s,r}^{(n)}
\end{equation}
\begin{equation}
B \cequiv \inlinematrix{\boxed{B^{(1)}} & & 0 \\ & \boxed{B^{(2)}} & & \\ & & \boxed{B} & \\ 0 & & & \boxed{B^{(4)}}} \rightarrow \text{kann diagonalisiert werden in jedem Kästchen}
B \cequiv \inlinematrix{\boxed{B^{(1)}} & & 0 \\ & \boxed{B^{(2)}} & & \\ & & \boxed{B^{(3)}} & \\ 0 & & & \boxed{B^{(4)}}} \rightarrow \text{kann diagonalisiert werden in jedem Kästchen}
\end{equation}
Falls $B^{(n)}$ entartet, gibt es einen dritten Opertor $C$ mit $[A,C] = [B,C] = 0$.
\end{description}

View File

@ -60,20 +60,20 @@ entsprechend
\begin{equation}
E_0 = \hbar \omega \left( n + \frac{1}{2} \right) + \lambda \dirac{n}{x^4}{n}
\end{equation}
%\begin{figure}[H] \centering
%\includegraphics{pdf/III/02-01-00.pdf}
%\end{figure}
\begin{figure}[H] \centering
\includegraphics{pdf/IV/02-01-00.pdf}
\end{figure}
Konsequenz?
\begin{itemize}
\item für $\lambda$ negativ $\abs{\lambda} \ll 1$
%\begin{figure}[H] \centering
%\includegraphics{pdf/III/02-01-01.pdf}
%\caption{gestrichelte Kurve entspricht $\frac{m}{2} \omega x^2 \abs{\lambda} x^4$}
%\end{figure}
\begin{figure}[H] \centering
\includegraphics{pdf/IV/02-01-01.pdf}
\caption{gestrichelte Kurve entspricht $\frac{m}{2} \omega x^2 \abs{\lambda} x^4$}
\end{figure}
\item für $\lambda$ positiv
%\begin{figure}[H] \centering
%\includegraphics{pdf/III/02-01-02.pdf}
%\end{figure}
\begin{figure}[H] \centering
\includegraphics{pdf/IV/02-01-02.pdf}
\end{figure}
\end{itemize}
volle Rechnung zeigt:
\begin{equation}
@ -89,9 +89,9 @@ mit
\item Obige Formel wegen Energienenner nicht anwendbar bei Entartung.
\item sehr relevant: Aufhebeung der Entartung durch Störung
\end{itemize}
%\begin{figure}[H] \centering
%\includegraphics{pdf/III/02-02-00.pdf}
%\end{figure}
\begin{figure}[H] \centering
\includegraphics{pdf/IV/02-02-00.pdf}
\end{figure}
\paragraph{Ansatz}
\begin{align}
\ket{a} &= \sum_{\alpha \in D} c_\alpha \ket{\alpha} + \sum_{\mu \notin D} d_\mu \ket{\mu}\\[10pt]
@ -140,7 +140,7 @@ Störung: $H_1$ sei E'feld
\begin{itemize}
\item in Ordnung $\lambda$ d.h. in $O(\abs{E})$:
\begin{equation}
\dirac{1,0,0}{H_1}{1,0,0} = \intgru{\Phi^*_{1,0,0} (\vec{r}) \cdor e \abs{E} z \Phi_{1,0,0}(\vec{r})}{r^3} = 0
\dirac{1,0,0}{H_1}{1,0,0} = \intgru{\Phi^*_{1,0,0} (\vec{r}) \cdot e \abs{E} \hat{z} \Phi_{1,0,0}(\vec{r})}{r^3} = 0
\end{equation}
\item in Ordnung $\abs{E}^2$:
\begin{align}
@ -179,11 +179,11 @@ diagonalisieren: $\dirac{\beta}{H_1}{\alpha}$
\begin{enumerate}
\item Diagonalelemente
\begin{equation}
\dirac{\alpha}{H_1}{\alpha} \tilde \intgru{\cos\theta\abs{\Phi_{n,l,m}}^2}{(\cos\theta)} \equiv 0
\dirac{\alpha}{H_1}{\alpha} \sim \intgru{\cos\theta\abs{\Phi_{n,l,m}}^2}{(\cos\theta)} \equiv 0
\end{equation}
\item Nichtdiagonalelemente
\begin{equation}
\dirac{n',l',m'}{\hat{z}}{n,l,m} \tilde \intgru{e^{i(m-m')\phi}}{\phi} \tilde \krondelta{m,m'}
\dirac{n',l',m'}{\hat{z}}{n,l,m} \sim \intgru{e^{i(m-m')\phi}}{\phi} \sim \krondelta{m,m'}
\end{equation}
mit
\begin{equation}

192
kapIV-2.tex.backup Normal file
View File

@ -0,0 +1,192 @@
\chapter{Zeitabhängige Störungstheorie}
Für $H = H_0 + \lambda_1$ mit $H_0$ exakt lösbar und $\lambda \ll 1$, lässt sich das Problem perturativ angeben.
\section{Nichtentarteter Fall}
gegeben:
\begin{equation}
\left(H_0 - E_\alpha \right) \ket{\alpha} = 0 ~ \left[ H_0 \ket{\alpha} = E_\alpha \ket{\alpha} \right]
\end{equation}
suche:
\begin{equation}
\left( H - E_a \right) \ket{a} = 0 \label{stern00}
\end{equation}
mit $\ket{a} \rightarrow \ket{\alpha}$ (für $x \rightarrow 0$) eindeutig, da nicht entartet.
\paragraph{Strategie} Wir entwickeln nach $\lambda$
\begin{equation}
\ket{a} = c_\alpha \ket{\alpha} + \sum_{\beta \neq \alpha} d_\beta \ket{\beta} \text{ und } d_\beta = O(\lambda)
\end{equation}
Norm:
\begin{align}
\abs{c_\alpha}^2 + \sum_{\beta \neq \alpha} \abs{d_\beta}^2 &= 1\\
\rightarrow c_\alpha &= 1 - O(\lambda^2)
\end{align}
einsetzen in (\ref{stern00}):
\begin{align}
0 &= \left( H - E_\alpha \right) \ket{\alpha}\\
0 &= \left( H_0 - \lambda H_1 - E_\alpha \right) \left( c_\alpha \ket{\alpha} + \sum_{\beta \neq \alpha} d_\beta \ket{\beta} \right) &\left| \bra{\gamma} ~ \gamma \neq \alpha \right.\\
&= c_\alpha \lambda \dirac{\gamma}{H_1}{\alpha} + \sum_{\beta \neq \alpha} d_\beta \left( E_\beta \krondelta{\beta,\gamma} + \lambda \dirac{\gamma}{H_1}{\beta} - E_a \krondelta{\beta,\gamma} \right)\\
0 &= \underbrace{c_\alpha}_{1-O(\lambda^2)} \lambda \dirac{\gamma}{H_1}{\alpha} + d_\gamma \left( E_\gamma - E_a \right) + \underbrace{\lambda \sum_{\beta \neq \alpha} d_\beta \dirac{\gamma}{H_1}{\beta}}_{O(\lambda^2)}
\end{align}
in niedrigster Ordnung erhält man:
\begin{align}
\rightarrow d_\gamma &= \lambda \frac{\dirac{\gamma}{H_1}{\alpha}}{\underbrace{E_a}_{E_\alpha - O(\lambda)} - E_\gamma} + O(\lambda^2)\\
\rightarrow d_\beta &= \lambda \frac{\dirac{\beta}{H_1}{\alpha}}{E_\alpha - E_\beta}\\
\ket{a} = \ket{\alpha} + \lambda \sum_{\beta \neq \alpha} \frac{\dirac{\beta}{H_1}{\alpha}}{E_\alpha - E_\beta} \ket{\beta} + O(\lambda^2)
\end{align}
gut falls:
\begin{equation}
\lambda \frac{\dirac{\beta}{H_1}{\alpha}}{E_\alpha - E_\beta} \ll 1 ~ \forall \beta \neq \alpha
\end{equation}
Energieverschiebung:
\begin{align}
0 &= \left( H - E_\alpha \right) \ket{a} &\left| \ket{\alpha} \right.\\
0 &= \bra{\alpha} \left( H_0 + \lambda_1 H_1 - E_a \right) \left( \ket{\alpha} + \lambda \sum_{\beta \neq \alpha} \frac{\dirac{\beta}{H_1}{\alpha}}{E_\alpha - E_\beta} \ket{\beta} + O(\lambda^2) \right)\\
E_a &= E_ \alpha + \lambda \dirac{\alpha}{H_1}{\alpha} + \lambda^2 \sum_{\beta \neq \alpha} \frac{\dirac{\alpha}{H_1}{\beta} \dirac{\beta}{H_1}{\alpha}}{E_\alpha - E_\beta} + O(\lambda^3)
\end{align}
\paragraph{Fazit}
\begin{enumerate}
\item In der führenden Ordnung ist die Energieverschiebung das Matrixelement der Störung im ungestörten Zustand.
\item Falls $\dirac{\alpha}{H_1}{\alpha}$ aus Symmetriegründen verschwindet, dann tragen alle Zustände zu nichtverschwindenden Korrekturen bei!
\end{enumerate}
\paragraph{Beispiel}
\begin{align}
H &= \frac{p^2}{2m} + \frac{m}{2} \omega^2 x^2 + \lambda \underbrace{x^4}_{H_1}\\
H_0 \ket{n} &= \hbar \omega \left( n + \frac{1}{2} \right) \ket{n}
\end{align}
Grundzustandsverschiebung
\begin{equation}
E_0 = \hbar \omega \frac{1}{2} + \lambda \dirac{0}{x^4}{0}
\end{equation}
entsprechend
\begin{equation}
E_0 = \hbar \omega \left( n + \frac{1}{2} \right) + \lambda \dirac{n}{x^4}{n}
\end{equation}
%\begin{figure}[H] \centering
%\includegraphics{pdf/III/02-01-00.pdf}
%\end{figure}
Konsequenz?
\begin{itemize}
\item für $\lambda$ negativ $\abs{\lambda} \ll 1$
%\begin{figure}[H] \centering
%\includegraphics{pdf/III/02-01-01.pdf}
%\caption{gestrichelte Kurve entspricht $\frac{m}{2} \omega x^2 \abs{\lambda} x^4$}
%\end{figure}
\item für $\lambda$ positiv
%\begin{figure}[H] \centering
%\includegraphics{pdf/III/02-01-02.pdf}
%\end{figure}
\end{itemize}
volle Rechnung zeigt:
\begin{equation}
E_n(\lambda) = \hbar \omega \left( n + \frac{1}{2} \right) + A^{(1)}_n (\lambda) + A^{(2)}_n (\lambda)
\end{equation}
mit
\begin{equation}
A^{(1)}_n (\lambda) = O(\lambda) ~ ; ~ A^{(2)}_n (\lambda) = O \left(e^{-\frac{1}{\lambda}} \right)
\end{equation}
\section{Entarteter Fall}
\begin{itemize}
\item Obige Formel wegen Energienenner nicht anwendbar bei Entartung.
\item sehr relevant: Aufhebeung der Entartung durch Störung
\end{itemize}
%\begin{figure}[H] \centering
%\includegraphics{pdf/III/02-02-00.pdf}
%\end{figure}
\paragraph{Ansatz}
\begin{align}
\ket{a} &= \sum_{\alpha \in D} c_\alpha \ket{\alpha} + \sum_{\mu \notin D} d_\mu \ket{\mu}\\[10pt]
0 &= \left( H - E_a \right) \ket{a}\\
\rightarrow 0 &= \left( H_0 + \lambda H_1 - E_a \right) \left( \sum_{\mu \notin D} d_\mu \ket{\mu} \right)
\end{align}
\paragraph*{Projektion auf $\bra{\nu} \notin D$}
\begin{align}
0 &= \lambda \sum_\alpha c_\alpha \dirac{\nu}{H_1}{\alpha} d_\nu \left( E_\nu - E_a \right) + \lambda \sum_{\mu \notin D} d_\mu \dirac{\nu}{H_1}{\mu}\\
\rightarrow d_\nu &= \lambda \frac{\sum_\alpha c_a \dirac{\nu}{H_1}{\alpha}}{\underbrace{E_a}_{E_\alpha} - E_\nu} + o(\lambda^2)
\end{align}
\paragraph*{Projektion auf $\bra{\beta} \in D$}
\begin{align}
0 &= \sum_\alpha c_\alpha \left( E_\alpha - E_a \right) \krondelta{\alpha,\beta} + \lambda \sum_\alpha c_\alpha \dirac{\beta}{H_1}{\alpha} + \lambda \sum_\mu d_\mu \dirac{\beta}{H_1}{\alpha}\\
0 &= \sum_\alpha c_\alpha \left( \left( E_a - E_\alpha \right) \krondelta{\alpha,\beta} + \underbrace{\lambda \dirac{\beta}{H_1}{\alpha} + \lambda^2 \sum_{\mu \notin D} \frac{\dirac{\beta}{H_1}{\mu} \dirac{\mu}{H_1}{\alpha}}{E_\alpha E_\mu}}_{\equiv \dirac{\beta}{H_\text{eff}}{\alpha}} \right) + O(\lambda^3)
\end{align}
$\forall \beta = 1, ..., N$ mit $N$ die Dimension von $D$\\
d.h. LGS für die $c_\alpha$ hat nichttriviale Lösung falls
\begin{equation}
\detb{E_\alpha \krondelta{\alpha,\beta} + \dirac{\beta}{H_\text{eff}}{\alpha} - E_\alpha \krondelta{\alpha,\beta}} = 0
\end{equation}
d.h. wir müssen $H_0 + H_\text{eff}$ im Unterraum $D$ diagonalisieren
\paragraph{Fazit}
\begin{enumerate}
\item in Ordnung $\lambda$ reicht es $H_1$ im entarteten Unterraum zu diagonalisieren
\item falls $H_1$ die Entartung nicht aufhebt, muss der $\lambda^2$-Term mitgenommen werden
\end{enumerate}
\section{Beispiel: Stark-Effekt}
H-Atom:
\begin{equation}
H_0 \ket{n,l,m} = E_n \ket{n,l,m}
\end{equation}
mit
\begin{equation}
E_n = \frac{l^2}{2 a_0} \frac{1}{n^2} \text{; Entartung } g(n) = n^2
\end{equation}
Störung: $H_1$ sei E'feld
\begin{equation}
H_1 = e \abs{E} \hat{z} \text{ für } \lambda = 1
\end{equation}
\paragraph{Erster Grundzustand}
\begin{equation}
\ket{1,0,0} \rightarrow \braket{r,\theta,\varphi}{1,0,0} = \Phi_{1,0,0}(\vec{r}) = \frac{U_{1,0}}{r}Y_{0,0} = \frac{1}{\sqrt{\pi} a_0^\frac{3}{2}}e^{-\frac{r}{a_0}}
\end{equation}
\begin{itemize}
\item in Ordnung $\lambda$ d.h. in $O(\abs{E})$:
\begin{equation}
\dirac{1,0,0}{H_1}{1,0,0} = \intgru{\Phi^*_{1,0,0} (\vec{r}) \cdor e \abs{E} z \Phi_{1,0,0}(\vec{r})}{r^3} = 0
\end{equation}
\item in Ordnung $\abs{E}^2$:
\begin{align}
E_a = E_\alpha + ... &= E_1 + \sum_{\beta,\alpha} \frac{\abs{\dirac{\beta}{H_1}{\alpha}}^2}{E_\beta - E_\alpha}\\
&= \left( E_1 \sum_{n=2}^\infty \sum_{l=0}^{n-1} \sum_{m=-l}^{+l} \frac{\abs{\dirac{n,l,m}{H_1}{1,0,0}}^2}{E_1 - E_n} + \int \text{Kontinuum} \right) + O(E^3)
\end{align}
mit
\begin{align}
\dirac{n,l,m}{\hat{z}}{1,0,0} &= \intgr{0}{-\infty}{r \intgr{-1}{+1}{\intgr{0}{2\pi}{\frac{U_{n,r}(r)}{r} Y^*_{l,m}(\theta,\varphi) r \underbrace{\cos\theta}_{Y_{1,0} \frac{1}{\sqrt{\pi} a_0^\frac{3}{2}}e^{-\frac{r}{a_0}}}}{\varphi}}{(\cos\theta)}}{r}\\
&= \krondelta{m,0} \krondelta{l,1} \frac{1}{\sqrt{3}} \int U_{n,1}(r) r U_{1,0}(r)
\end{align}
\end{itemize}
am meisten trägt $n=2$ bei
\begin{align}
\dirac{2,1,0}{\hat{z}}{1,0,0} &= \frac{a_0}{\sqrt{3}} \intgr{0}{\infty}{\frac{r^2 e^{-\frac{r}{2}}}{2 \sqrt{6}} r \left( 2r e^{-r} \right)}{r}\\
&= \frac{2^7 \sqrt{2}}{3^5} a_0
\end{align}
und damit ist
\begin{equation}
E_{1,0,0} = E_1 - 1,48 a_0^3 \abs{E}^2
\end{equation}
und für alle weiteren $n$
\begin{equation}
E_{1,0,0} \rightarrow E_1 - \frac{9}{4} a_0^3 \abs{E}^2
\end{equation}
\paragraph{linerarer Stark-Effekt für n=2}
\begin{align}
\braket{r,\theta,\varphi}{2,0,0} &= \Phi_{2,0,0}(\vec{r}) = \frac{1}{\sqrt{2 a_0^3}} \left(1 - \frac{r}{2a_0} \right) e^{-\frac{r}{2a_0}} Y_{0,0}\\
\braket{r,\theta,\varphi}{2,1,m} &= \Phi_{2,1,m} (\vec{r}) = \frac{1}{24 a_0^3} \frac{r}{a_0} e^{-\frac{r}{2a_0}} Y_{l,m}
\end{align}
im entartetetn Unterraum $\setCond{\bra{\alpha}}{\alpha = 1,2,3,4}$ mit
\begin{equation}
\bra{1} = \bra{1,0,0} ;~ \bra{2} = \bra{2,1,0} ;~ \bra{3} = \bra{2,1,1} ;~ \bra{4} = \bra{2,1,-1}
\end{equation}
diagonalisieren: $\dirac{\beta}{H_1}{\alpha}$
\begin{enumerate}
\item Diagonalelemente
\begin{equation}
\dirac{\alpha}{H_1}{\alpha} \tilde \intgru{\cos\theta\abs{\Phi_{n,l,m}}^2}{(\cos\theta)} \equiv 0
\end{equation}
\item Nichtdiagonalelemente
\begin{equation}
\dirac{n',l',m'}{\hat{z}}{n,l,m} \tilde \intgru{e^{i(m-m')\phi}}{\phi} \tilde \krondelta{m,m'}
\end{equation}
mit
\begin{equation}
\dirac{1}{H_1}{2} = \dirac{2}{H_1}{1} = \dirac{2,0,0}{e\abs{E}z}{×}
\end{equation}
\end{enumerate}

69
kapIV-3.tex Normal file
View File

@ -0,0 +1,69 @@
\chapter{Bohr'sche Näherung für Streutheorie}
% \begin{figure}[H] \centering
% \includegraphics{pdf/IV/03-00-00.pdf}
% \end{figure}
\section{Geometrie}
% \begin{figure}[H] \centering
% \includegraphics{pdf/IV/03-01-00.pdf}
% \end{figure}
\section{Stat SG}
\equationblock{\Phi\sbk{\vec{r}} = \Phi^\text{in}\sbk{\vec{n}} + \Phi^\text{ex}\sbk{\vec{r}}}
mit
\equationblock{\Phi^\text{ex}\sbk{\vec{r}} \longrightarrow^{\vec{r}\rightarrow\infty} f\sbk{\theta,\Phi} \frac{e^{\i k \vec{r}}}{r}}
und
\equationblock{\Phi^\text{in}\sbk{\vec{r}} = e^{i \vec{k} \vec{r}}}
SG:
\begin{align}
\sbk{-\frac{\hbar^2}{2 \mu} \vec{\nabla}^2 + V\sbk{\vec{r}}} \Phi\sbk{\vec{r}} &= E \Phi\sbk{\vec{r}} &\left| -\frac{2 \mu}{\hbar^2} \right. \\
\sbk{\vec{\nabla}^2 - V\sbk{\vec{r}}} \Phi\sbk{\vec{r}} &= -\frac{2 \mu}{\hbar^2} E \Phi\sbk{\vec{r}} &\left; E = \frac{\hbar^2}{2 \mu} k^2 \right. \\
\rightarrow \sbk{\vec{\nabla}^2 + k^2} \Phi\sbk{\vec{r}} &= V\sbk{\vec{r}} \Phi\sbk{r}
\end{align}
Green's Funktion Ansatz
\begin{align}
&\sbk{\vec{\nabla}^2 + k^2} G^0\sbk{\vec{r}-\vec{r'}} = \delta\sbk{\vec{r}-\vec{r'}} \\
&\Phi\sbk{\vec{r}} = \intgru{}{r'} G^\sbk{0}\sbk{\vec{r}-\vec{r'}} V\sbk{\vec{r'}} \Phi\sbk{\vec{r'}} + \underbrace{\Phi^\sbk{0}\sbk{\vec{r}}}_{\text{beliebige Lösung der homogenen Gl.}}
\end{align}
\begin{enumerate}
\item für $V = 0$: \equationblock{\Phi\sbk{\vec{r}} = \Phi^\sbk{0}\sbk{\vec{r}} = e^{\i \vec{k} \vec{r}}}
\item in $\bigOb{V}$: \equationblock{\Phi{\vec{r}} \approx \sbk{\intgru{}{\vec{r}} G^\sbk{0}\sbk{\vec{r}-\vec{r'}} v\sbk{\vec{r}} e^{\i \vec{k} \vec{r}}} + e^{\i \vec{r} \vec{r}}}
\item im Prinzip iterieren:
\begin{align}
\Phi &= \Phi^0 + \G^\sbk{0}v \Phi^0 + G^\sbk{0} v G^\sbk{0} \Phi^\sbk{0} + \ldots
&= \frac{1}{1 - G^\sbk{0} v} \Phi^\sbk{0} &\left( geometrische Reihe \right)
\end{align}
formal exakt, praktische ziemlich nutzlos
\end{enumerate}
\section{Berechnung der Green'schen Funktion}
\begin{align}
\sbk{\vec{\nabla}^2 + k^2} G\sbk{\vec{u}} &= \delta\sbk{\vec{u}} &\left| \intgru{e^{-\i \vec{q} \vec{u}}}{\vec{u}} \right. \\
\sbk{-\vec{q}^2 + k^2} G\sbk{q} &= 1 \\
G\sbk{q} &= \frac{1}{k^2 - q^2} \\
G\sbk{\vec{ u}} &= \intgru{\frac{1}{\sbk{2 \pi}^2} \frac{1}{k^2 - q^2} e^{\i \vec{q} \vec{u}}}{q} \\
&= \frac{1}{4 \pi^2} \intgr{-1}{+1}{\intgr{0}{\infty}{q^2 \frac{1}{k^2 - q^2} e^{\i q u \cosb{\theta}}}{q}}{\sbk{\cosb{\theta}}} \\
&= \frac{1}{4 \pi^2} \intgr{0}{\infty}{\frac{q^2}{\i q u} \sbk{e^{\i q u} - e^{-\i q u}} \frac{1}{k^2 - q^2}}{q} \\
&= \frac{1}{4 \pi^2} \frac{1}{\i n} \intgrinf{\frac{q e^{\i q n}}{k^2 -q^2}}{q}
&= \frac{1}{4 \pi^2} \frac{1}{\i n} \intgrinf{\frac{q e^{\i q u}}{k^2 - q^2 + \i \epsilon}}{q} \\
\text{Residuensatz} \Rightarrow &= \frac{1}{4 \pi^2} \frac{1}{\i u} 2 \pi \i \underbrace{Res\sbk{q=k}_{\epsilon \rightarrow 0}}_{-\frac{k}{2 k} e^{\i k u} \\
G\sbk{u} &= -\frac{1}{4 \pi u} e^{\i k u}
\end{align}
% \begin{figure}[H] \centering
% \includegraphics{pdf/IV/03-03-00.pdf}
% \end{figure}
\section{Bohr'sche Näherung}
\equationblock{}
\section{Streuamplitude und differentieller Wirkungsquerschnitt}
Bsp.: abgeschirmtes Coulomb-Potential
Yukawa Potential
\begin{align}
V\sbk{r} &= \frac{l^2}{r} l^{-\frac{r}{r_0}} \\
V\sbk{q} &= \frac{4 \pi l^2}{q}\intgr{0}{\infty}{\sinb{q r'} l^{-\frac{r}{r_0}}}{r'} \\
&= 4 \pi l^2 \frac{1}{q^2 + \frac{1}{r_0^2}} \\
\diffTfrac{r}{\Omega} &= \frac{l^2}{4 \mu^2 \tilde{V}^4 \sin^4\sbk{\frac{\theta}{2}}} &\left \tilde{V} = \frac{\hbar k}{\mu} \left.
\end{align}
Rutherford'sche Streuquerschnitt für das Coulomb-Problem

163
kapV-1.tex Normal file
View File

@ -0,0 +1,163 @@
\chapter{Reine Zustände}
\section{Postulate}
\begin{itemize}
\item P1: Bei vollständiger Kenntnis (Präparation) wird ein System durch einen normierten Vektor
\equationblock{\ket{\Psi} \in \hilbert} beschrieben
\item P2a: Jeder physikalischen Größe entspricht ein hermitescher Operator
\equationblock{A = \sum a_n \ket{n} \bra{n} \text{(Spektraldarstellung)}}
mit Eigenzuständen $\ket{n}$ und reellen Eigenwerten $a_n$
\equationblock{A = \sum a_n \ket{n} \bra{n} = \sum_\nu a_\nu P_\nu}
mit $P_\nu = \sum_{n : a_n = a_\nu} \ket{n} \bra{n}$
\item P2b: Eine Messung von A im Zustand $\ket{\Psi}$ gibt Sicherheit einen der Eigenzustände $a_\nu$
Die Wahrscheinlichkeit, $a_\mu$ zu messen ist:
\begin{align}
\probb{A \cequiv a_\mu}{\ket{\Psi_0}} &= \dirac{\Psi_0}{P_\mu}{\Psi_0} \\
&= \braket{\Psi}{n} \braket{m}{\Psi} \\
&= \spbk{\braket{m}{\Psi}}^2 \\
&= \sum_k \dirac{\Psi_0}{P_\mu}{k} \braket{k}{\Psi_0} \\
&= \sum_k \braket{k}{\Psi_0} \dirac{\Psi_0}{P_\mu}{k} \\
&= \tr\sbk{\ket{\Psi_0} \bra{\Psi_0} P_\mu} \\
&= \tr\sbk{P_{\Psi_0} P_\mu}
\end{align}
mit $P_{\Psi_{0}} = \ket{\Psi_0} \bra{\Psi_0}$ \\
Konsequenz: ``Erwarutngswert'' oder Mittelwert über viele Messungen in identisch präparierten Zustand $\ket{\Psi_0}$
\begin{align}
\expval{A_{\Psi_0}} &= \sum_\nu a_\nu \probb{A \cequiv a_\nu}{\ket{\Psi_0}} \\
&= \sum_\nu a_\nu \dirac{\Psi_0}{P_\nu}{\Psi_0} \\
&= \dirac{\Psi_0}{A}{\Psi_0}
\end{align}
\item P2c: Unmittelbar nach der Messung des Messwertes $a_\mu$ ist das System im Zustand
\equationblock{\ket{\Psi} = \frac{O_\mu \ket{\Psi_0}}{\norm{P_\mu} \ket{\Psi_0}} \stackrel{a_\mu nicht entartet}{} \ket{m}}
\item P3: Nach einer Messung oder Präparation entwickelt sich der Zustand nach der Schrödingergleichung:
\equationblock{\i \hbar \diffPs{t} \ket{\Psi_0\sbk{t}} = H\sbk{t} \ket{\Psi\sbk{t}}}
mit dem (hermiteschen) Hamiltonoperator $H\sbk{t}$
\end{itemize}
\section{Einfaches Beispiel mit Spin $\frac{1}{2}$}
% \begin{figure}[H] \centering
% \includegraphics{pdf/V/01-02-00.pdf}
% \end{figure}
\begin{itemize}
\item P1: $\hilbert = \setC^2$ \\
Basis: $\ket{z+}$, $\ket{z-}$ \\
allgemeiner Zustand: $\ket{\Psi} = c_1 \ket{z+} + c_2 \ket{z-}$ mit $\spbk{c_1}^2 + \spbk{^2} = 1$
\item P2a: Mögliche physikalische Größen: Messung durch SG in $\vec{n}$ Richtung:
% \begin{figure}[H] \centering
% \includegraphics{pdf/V/01-02-01.pdf}
% \end{figure}
mögliche Messwerte: Eigenwerte von $V_n = \pm 1$ \\
Eigenvektoren $\ket{n+} = \inlinematrixu{\cosb{\frac{\Theta}{2}} \\ e^{\i \Phi} \sinb{\frac{\Theta}{2}}}$
$n = \inlinematrixu{\sinb{\Theta}\cosb{\Phi} \\ \sinb{\Theta}\sinb{\Phi} \\ \cosb{\Theta}}$
\item P2b: $\probb{\Sigma_n \cequiv +1}{\ket{\Psi_0}} = \spbk{\braket{n+}{\Psi_0}}^2$
\item P2c: Nach der Messung von +1 mit Sicherheit im Zustand $\ket{n+}$ \\
Besipiel für den Erwartungswert:
\begin{align}
\expval{\Sigma_n}_\ket{\Psi_0} &= \\
\dirac{\Psi_0}{\Sigma_n}{\Psi_0} &=
\inlinematrixu{c_1^\ast & c_2} \inlinematrixu{n_z & n_x - \i n_y \\ n_x + \i n_y & -n_z} \inlinematrixu{c_1 \\ c_2}
\end{align}
\item P3: Dynamik im Magnetfeld:
\equationblock{H\sbk{t} = - \vec{mu} \vec{B}\sbk{t} = g \mu_b \frac{1}{2} \vec{\Sigma} \cdot \vec{B}\sbk{t}}
Beispiel: $\vec{B}\sbk{t} ) B_z \vec{e_z} \rightarrow H = \frac{\hbar \omega}{2} \Sigma_z$
mit $\omega = g \mu_b \frac{B}{\hbar}$
$\text{SG(P3)} = \i \hbar \diffPs{t} \ket{\Psi} = \hbar \frac{\hbar}{2} \Sigma_z \ket{\Psi}$
$\Psi\sbk{t} = e^{-\frac{\i}{\hbar} H t} \ket{\Psi_0} =$ \\
$c_1\sbk{0} e^{-\frac{\i \omega t}{2}} \ket{z+} + c_2\sbk{0} e^{+\frac{\i \omega t}{2}} \ket{z-}$
\end{itemize}
\chapter{Gemische: Statistischer Operator}
\section{Motivation: Ein Spiel}
% \begin{figure}[H] \centering
% \includegraphics{pdf/V/02-01-00.pdf}
% \end{figure}
Alice sendet Bob mit Wahrscheinlichkeit $p_+ = \frac{1}{2}$ den Zustand $\ket{z+}$ und mit $p_- = \frac{1}{2}$ den Zustand $\ket{z-}$
Bob weiss nicht ob Alice $\ket{z+}$ oder $\ket{z-}$ geschicket hat. Bob darf aber ein beliebiges Stern-Gerlach-Experiment durchführen.
Frage: Wie soll Bob seinen Einganszustand beschreiben?
\begin{enumerate}
\item Bobs Experiment(e) zeigen: \\
$\expval{\Sigma_z} = \expval{\Sigma_x} = \expval{\Sigma_x} = \expval{\Sigma_n}$ \\
Es gibt kein $\ket{\Psi_0}$ mit $\expval{\Sigma_n}_\ket{\Psi} = 0 \forall n$ \\
Bobs Kenntnis ist unvollständig (Eingangspräparation)
\item Bobs Input besteht aus einem \textbf{klassischen} Ensemble (Gesamtheit), in dem mit \textbf{klassischer} Wahrscheinlichkeit $p_+ = \frac{1}{2}$ der Zustand $\ket{z+}$ und mit \textbf{klassischer} Wahrscheinlichkeit $p_- = \frac{1}{2}$ der Zustand $\ket{z-}$ enthalten ist.
\end{enumerate}
\section{Definition des statistischen Operators (Dichtematrix; engl. density matrix)}
Sei $\rho$ ein Operator $\hilbert \rightarrow \hilbert$:
\begin{enumerate}
\item $\rho = \rho^\dagger$
\item $\tr\sbk{\rho} = 1$
\item $\dirac{\psi}{\rho}{\psi} \geq 0 \forall \ket{\psi} \in \hilbert$
\end{enumerate}
bzw. in irgend einer Basis $\sgbk{\ket{n}}$
\begin{enumerate}
\item $\rho_{nn} = \dirac{n}{\rho}{m} \rho^\ast_{mn}$
\item $\sum_n \rho_{nn} = 1$
\item $\sum_{n,m} c_n^\ast \rho_{nm} c_m \geq \forall c_n$ mit $\sum_n \spbk{c_n}^2$
\end{enumerate}
\section{Gemisch}
Definition: Ein quantales enthält mit Wahrscheinlichkeit $p_i$ den reinen Zustand $\ket{\psi_i}$ $\sbk{i = 1\ldots M}$ $M$ beliebig, im Allgemeinen ist $M \neq \dim \hilbert$.
\begin{enumerate}
\item $\sum_i^M p_i = 1$
\item $\braket{\psi_i}{\psi_j} \neq \delta_{ij}$ erlaubt
\end{enumerate}
Dieses quantale Gemisch wird durch den statistischen Operator $\rho = \sum_{i=1}^M p_i \ket{\psi_i}\bra{\psi_i} = \sum_{i=1}^M p_i P_{\psi_i}$.
Alice präpariert $\ket{z+}$ und $\ket{z-}$ Zustand, sie würfelt und wählt dann mit Wahrscheinlichkeit $\frac{1}{2}$ $\ket{z+}$ und mit Wahrscheinlichkeit $\frac{1}{2}$ $\ket{z-}$, die sie zu Bob schickt.
Frage: Wie soll Bob den Eingangszustand beschreiben?
Bobs mögliche Messwerte $\sigma_n$ sind immer noch $\pm1$.
\begin{align}
\prob{\sigma_z \cequiv +1} &= p_{z+} &= \frac{1}{2} \\
\prob{\sigma_z \cequiv -1} &= p_{z-} &= \frac{1}{2} \\
\prob{\sigma_n \cequiv +1} &= p_{z+} &= p_{z-} \probb{\sigma_z \cequiv +1}{\ket{z+}} + p_{z-} \probb{\sigma_z \cequiv -1}{\ket{z+}}
\end{align}
Bsp: $\vec{n} = \vec{e_x}$
\begin{align}
\prob{\sigma_x \cequiv 1} &= \frac{1}{2} \cdot \frac{1}{2} + \frac{1}{2} \cdot \frac{1}{2} &= \frac{1}{2} \\
\prob{\sigma_x \cequiv -1} &= &= \frac{1}{2}
\end{align}
$\Rightarrow \ssbk{\sigma_x} = 0$
Check:
\begin{enumerate}
\item $\rho^\dagger = \sum_i p_i \ket{\psi_1} \bra{\psi_1} = \rho$
\item \begin{align}
\tr\sbk{\rho} &= \sum_i \dirac{n}{\sum_{i=1}^M P_i}{\psi_i} \braket{\psi_i}{n} \\
&= \sum_{i=1}^M P_i \spbk{\underbrace{\braket{n}{\psi_i}}_{=1}}^2 = 1
\end{align}
\item \begin{align}
\dirac{\psi}{\rho}{\psi} &= \dirac{\psi}{\sum_{i=1}^M P_i}{\psi_i} \braket{\psi_i}{\psi} \\
&= \sum_{i=1}^M P_i \ssbk{\underbrace{\braket{\psi_i}{\psi}}_{\geq 0}}^2
\end{align}
\end{enumerate}
Bemerkung:
\begin{enumerate}
\item Als Spezialfall enthält der Begriff Gemisch auch den reinen Zustand. $M=1$ gibt $\rho= \ket{\psi_1} \bra{\psi_1} = P_{\ket{\psi_1}}$
\item für einen reinen Zustand gilt: $\rho^2 = \rho$
\end{enumerate}
Beweis: $\rho^2 = \rho \cdot \rho = \ket{\psi_1} \braket{\psi_1}{\psi_1} \bra{\psi_1} = \ket{\psi_1} \bra{\psi_1} = \rho$
Beispiel:
Alice präpariert mit Wahrscheinlichkeit $p_1$ den Zustand $\ket{z+}$ und mit $p_2$ den Zustand $\ket{x+}$ $\sbk{p_1 + p_2 = 1}$
\begin{align}
\rho &= p_1 \ket{z+} \bra{z+} + p_2 \ket{x+} \bra{x+} \\
&= p_1 \inlinematrixu{1 \\ 0} \inlinematrixu{1 & 0} + p_2 \inlinematrixu{\frac{\sqrt{2 }}{2} \\ \frac{\sqrt{2}}{2}} \inlinematrixu{\frac{\sqrt{2}}{2} & \frac{\sqrt{2}}{2}} \\
&= p_1 \inlinematrixu{1 & 0 \\ 0 & 0} + p_2 \inlinematrixu{\frac{1}{2} & \frac{1}{2} \\ \frac{1}{2} & \frac{1}{2}} \\
&= \inlinematrixu{p_1 + \frac{p_2}{2} & \frac{p_2}{2} \\ \frac{p_2}{2} & \frac{p_2}{2}}
\end{align}
$\rho^2 = \rho \gdw p_1=1 \text{xor} p_1=0$
Bemerkung:
Die Darstellungen eines Gemisches eines

View File

@ -26,6 +26,7 @@
\newcommand{\im}[1]{{\text{Im}\left( #1 \right)}}
\newcommand{\tr}{{\text{tr}}}
\newcommand{\sign}{{\text{sign}}}
% \newcommand{\dim}{\text{dim}}
\newcommand{\QED}{\begin{large}\textbf{\checkmark}\end{large}}
\newcommand{\cequiv}{\stackrel{\scriptscriptstyle\wedge}{=}} %ContextEQUIvalent
@ -44,6 +45,8 @@
\newcommand{\sbk}[1]{\left( #1 \right)}
\newcommand{\sqbk}[1]{\left[ #1 \right]}
\newcommand{\ssbk}[1]{\left< #1 \right>}
\newcommand{\spbk}[1]{{\left| #1 \right|}}
\newcommand{\sgbk}[1]{\left\{ #1 \right\}}
\newcommand{\detb}[1]{\det\sqbk{#1}}
\newcommand{\cosb}[1]{\cos\sbk{#1}}
\newcommand{\sinb}[1]{\sin\sbk{#1}}
@ -60,5 +63,7 @@
\newcommand{\levicivita}[1]{\varepsilon_{#1}}
\newcommand{\krondelta}[1]{\delta_{#1}}
% \newcommand{\sum}[3]{\Sigma_{#1}^{#2} #3}
% \newcommand{\prod}[3]{\Pi_{#1}^{#2} #3}
\newcommand{\equationblock}[1]{\begin{equation} #1 \end{equation}}

View File

@ -1,295 +0,0 @@
[General]
img_extIsRegExp=false
img_extensions=.eps .jpg .jpeg .png .pdf .ps .fig .gif
kileprversion=2
kileversion=2.0
lastDocument=kapIV-2.tex
masterDocument=
name=Theo2
pkg_extIsRegExp=false
pkg_extensions=.cls .sty
src_extIsRegExp=false
src_extensions=.tex .ltx .latex .dtx .ins
[Tools]
MakeIndex=
QuickBuild=PDFLaTeX+ViewPDF
[item:formelsammlung.tex]
archive=true
column=115
encoding=UTF-8
highlight=LaTeX
line=42
open=false
order=6
[item:kapI-1.tex]
archive=true
column=17
encoding=UTF-8
highlight=LaTeX
line=103
open=false
order=3
[item:kapI-2.tex]
archive=true
column=13
encoding=UTF-8
highlight=LaTeX
line=163
open=false
order=4
[item:kapI-3.tex]
archive=true
column=0
encoding=
highlight=LaTeX
line=0
open=false
order=-1
[item:kapI-4.tex]
archive=true
column=6
encoding=UTF-8
highlight=LaTeX
line=192
open=false
order=5
[item:kapI-5.tex]
archive=true
column=7
encoding=UTF-8
highlight=LaTeX
line=148
open=false
order=6
[item:kapI-6.tex]
archive=true
column=4
encoding=
highlight=LaTeX
line=2
open=false
order=-1
[item:kapII-0.tex]
archive=true
column=12
encoding=UTF-8
highlight=LaTeX
line=122
open=false
order=1
[item:kapII-1.tex]
archive=true
column=113
encoding=UTF-8
highlight=LaTeX
line=67
open=false
order=-1
[item:kapII-2.tex]
archive=true
column=9
encoding=UTF-8
highlight=LaTeX
line=91
open=false
order=7
[item:kapII-3.tex]
archive=true
column=12
encoding=UTF-8
highlight=LaTeX
line=123
open=false
order=2
[item:kapII-4.tex]
archive=true
column=13
encoding=UTF-8
highlight=LaTeX
line=39
open=false
order=2
[item:kapII-5.tex]
archive=true
column=29
encoding=UTF-8
highlight=LaTeX
line=92
open=false
order=3
[item:kapIII-0.tex]
archive=true
column=8
encoding=UTF-8
highlight=LaTeX
line=0
open=false
order=4
[item:kapIII-1.tex]
archive=true
column=15
encoding=UTF-8
highlight=LaTeX
line=39
open=false
order=5
[item:kapIII-2.tex]
archive=true
column=20
encoding=UTF-8
highlight=LaTeX
line=1
open=false
order=6
[item:kapIII-3.tex]
archive=true
column=14
encoding=UTF-8
highlight=LaTeX
line=319
open=false
order=7
[item:kapIII-4.tex]
archive=true
column=0
encoding=UTF-8
highlight=LaTeX
line=195
open=false
order=8
[item:kapIV-1.tex]
archive=true
column=0
encoding=UTF-8
highlight=LaTeX
line=36
open=true
order=1
[item:kapIV-2.tex]
archive=true
column=0
encoding=UTF-8
highlight=LaTeX
line=209
open=true
order=2
[item:math.tex]
archive=true
column=17
encoding=UTF-8
highlight=LaTeX
line=46
open=false
order=2
[item:physics.tex]
archive=true
column=0
encoding=UTF-8
highlight=LaTeX
line=18
open=false
order=2
[item:theo2.kilepr]
archive=true
column=0
encoding=
highlight=
line=0
open=false
order=-1
[item:theo2.tex]
archive=true
column=0
encoding=UTF-8
highlight=LaTeX
line=47
open=true
order=0
[item:ueb1.tex]
archive=true
column=2147483647
encoding=
highlight=
line=0
open=false
order=-1
[item:ueb2.tex]
archive=true
column=64
encoding=
highlight=
line=0
open=false
order=-1
[item:ueb3.tex]
archive=true
column=2147483647
encoding=
highlight=
line=0
open=false
order=-1
[item:ueb4.tex]
archive=true
column=2147483647
encoding=
highlight=
line=0
open=false
order=-1
[item:ueb5.tex]
archive=true
column=0
encoding=
highlight=LaTeX
line=0
open=false
order=-1
[item:ueb6.tex]
archive=true
column=0
encoding=UTF-8
highlight=LaTeX
line=119
open=false
order=-1
[item:ueb7.tex]
archive=true
column=0
encoding=UTF-8
highlight=LaTeX
line=0
open=false
order=1

View File

@ -48,24 +48,30 @@
\include{kapIII-3}
\include{kapIII-4}
\part{Näherungsmethoden}
\label{IV}
\include{kapIV-1}
\include{kapIV-2}
% \part{Übungsmitschrieb}
% \label{UE}
% \include{ueb1}
% \include{ueb2}
% \include{ueb3}
% \include{ueb4}
% \include{ueb5}
% \include{ueb6}
% \include{ueb7}
% \include{ueb8}
% \include{ueb9}
% \include{ueb10}
% \include{ueb11}
\part{Übungsmitschrieb}
\label{UE}
\include{ueb1}
\include{ueb2}
\include{ueb3}
\include{ueb4}
\include{ueb5}
\include{ueb6}
\include{ueb7}
\include{ueb8}
\include{ueb9}
\include{ueb10}
\include{ueb11}
\part{Wiederholung}
\label{V}
\include{kapV-1}
\part{Formelsammlung}
\label{FS}